0% found this document useful (0 votes)
176 views5 pages

2011 U of I Mock Putnam Contest Solutions

This document contains solutions to problems from past Putnam math contests. The first solution proves that the position of the first digit of the number 10k is given by the formula f(10k) = k10k - (10k - 1)/9 + 1. The second solution proves that an is even if and only if n is odd, where an is the greatest integer of (√2 + 1)n. The third solution proves an inequality relating the nth roots of sums of products of positive real numbers. The fourth solution shows that a series involving distances between points in 3D space converges when the exponent is greater than 3 but diverges when the exponent is 3.

Uploaded by

Gag Paf
Copyright
© © All Rights Reserved
We take content rights seriously. If you suspect this is your content, claim it here.
Available Formats
Download as PDF, TXT or read online on Scribd
0% found this document useful (0 votes)
176 views5 pages

2011 U of I Mock Putnam Contest Solutions

This document contains solutions to problems from past Putnam math contests. The first solution proves that the position of the first digit of the number 10k is given by the formula f(10k) = k10k - (10k - 1)/9 + 1. The second solution proves that an is even if and only if n is odd, where an is the greatest integer of (√2 + 1)n. The third solution proves an inequality relating the nth roots of sums of products of positive real numbers. The fourth solution shows that a series involving distances between points in 3D space converges when the exponent is greater than 3 but diverges when the exponent is 3.

Uploaded by

Gag Paf
Copyright
© © All Rights Reserved
We take content rights seriously. If you suspect this is your content, claim it here.
Available Formats
Download as PDF, TXT or read online on Scribd
You are on page 1/ 5

2011 U OF I MOCK PUTNAM CONTEST

Solutions
1. [Variation of A2, Putnam 1987] The sequence of digits
1 2 3 4 5 6 7 8 9 1 0 1 1 1 2 1 3 ...
is obtained by writing out the natural numbers in order. Let f (n) denote the position of the first digit of the number
n in this sequence. Thus, for example, f (1) = 1, f (2) = 2, f (10) = 10 (since the integer n = 10 occupies positions 10
and 11 in this sequence), f (11) = 12 (since 11 occupies positions 12 and 13), f (12) = 14 (since 12 occupies positions
14 and 15, and so on.
Find, with proof, a simple explicit formula for f (10k ), where k is an arbitrary positive integer.
Solution. The desired formula is f (10k = k10k (10k 1)/9 + 1 . To prove this, note that 10k is the first integer
with k + 1 digits. Thus, the position of its first digit is 1 plus the total number of positions occupied by the integers
with at most k digits.
Now there are 10i 10i1 integers with exactly i digits (namely, all integers n in the range 10i1 n < 10i ), so the
total number of positions occupied by integers with i digits is i(10i 10i1 ). Adding these counts for i = 1, 2, . . . , k 1
gives the number of positions occupied by integers with at most k digits:
k
X k
X k1
X
(10i 10i1 )i = 10i i 10j (j + 1)
i=1 i=1 j=0
k1
X
= 10k k 10j
j=0

10k 1
= 10k k .
10 1
Adding 1 to this count gives the above formula for f (10k ).

2. Let an = [( 2 + 1)n ], where [x] denotes the greatest integer x (i.e., the floor function). Prove that an is even if and
only if n is odd.

Solution. Let bn := (1 + 2)n + (1 2)n . Expanding each of the two nth powers by the binomial theorem shows
that
n  
X n k k X n
bn = ( 2) + ( 2) = 2 2k/2 .
k k
k=0 k even
n
Hence bn is an even integer. Now (1 2) is a number of absolute
value at most 2 1 < 1/2 and is negative
for n
odd, and positive for n even. Since ( 2 + 1)n = bn (1 2)n , by the definition of bn , it follows that [( 2 + 1)n ] is
equal to bn when n is odd, and equal to bn 1 when n is even. Since bn is always even, this proves the claim.

3. [A2, Putnam 2003] Let a1 , . . . , an , b1 , . . . , bn be positive real numbers. Prove that


(a1 . . . an )1/n + (b1 . . . bn )1/n (a1 + b1 )1/n . . . (an + bn )1/n .

Solution. Dividing by the right-hand side, the given inequality can be rewritten as
(1) (x1 . . . xn )1/n + (y1 . . . yn )1/n 1,
where
ai bi
xi =
, yi = .
ai + bi ai + bi
Applying AGM to each term on the left of (1) gives
n n n
1X 1X 1X
(x1 . . . xn )1/n + (y1 . . . yn )1/n xi + yi = (xi + yi ) = 1,
n i=1 n i=1 n i=1

since xi + yi = 1 for each i.


4. Let P1 , P2 , P3 , . . . be a sequence of points in 3-dimensional space satisfying (i) |Pi | 1 for all i and (ii) |Pi Pj | 1
for all i and j with i 6= j. (Here |P Q| denotes the usual (Euclidean) distance between P and Q, and |P | denotes the
distance between P and the origin.)
(a) Prove that, if > 3, then the infinite series

X 1
i=1
|Pi |
converges.
(b) Show that there exists a sequence of points Pi satisfying conditions (i) and (ii) above for which the above series
diverges when = 3.
Solution. (a) First note that, without loss of generality, we may assume that the Pi all lie in the first octant. Subdivide
this octant into boxes of side length 1/2 of the form
     
m m+1 n n+1 p p+1
B(m, n, p) = , , , ,
2 2 2 2 2 2
p
where m, n, p are nonnegative integers. Since the largest distance between any two points in such a box is 3/4 < 1,
such a box can contain at most one of the points Pi . Moreover, since |Pi | 1, the box B(0, 0, 0) cannot contain a point
Pi .
Now note that, if Pi B(m, n, p), then
 m n p  1 p
|Pi | , , = m2 + n2 + p2 .

2 2 2 2
Hence

X 1 X 2

i=1
|Pi | m,n,p=0
(m2 + n2 + p2 )/2
max(m,n,p)>0

X 1
3!2 2 + n2 + p2 )/2
(using symmetry in m, n, p)
0mnp
(m
p6=0

X 1
6 2 (since m2 + n2 + p2 p2 )
p
0mnp
p6=0

X (p + 1)2
6 2 (since there are (p + 1)2 choices of (m, n) with m, n p)
p=1
p

X (2p)2
6 2 (since p + 1 2p)
p=1
p


X 1
= 24 2 ,
p=1
p2

and since > 3, the latter series converges. This proves part (a).
(b) If we let the Pi be the lattice points (m, n, p) with positive integral coordinates, then the conditions |Pi | 1 and

1
|Pi Pj | 1 are satisfied. On the other hand, we have

X 1 X 1

i=1
|Pi |3 m,n,p=1
(m2 + n2 + p2 )3/2

k+1 k+1 k+1


2X1 2 X1 2 X1
X 1

k=1 m=2k
(m2 + n2 + p2 )3/2
n=2k p=2k
k+1 k+1 k+1
2X1 2 X1 2 X1
X 1
(since (m2 + n2 + p2 )3/2 (3 22k+2 )3/2 100 23k )
100 23k
k=1 m=2k n=2k p=2k

1 X 1
= (2k )3 (since there are (2k )3 choices of (m, n, p))
100 23k
k=1

1 X
= 1 = ,
100
k=1
P
so the series i=1 1/|Pi |3 diverges.

5. Determine, with proof, whether the series



X 1
n 2+cos(2 ln n)
n=1
converges.
Solution. We claim that the series diverges (though only barely so). The proof is somewhat technical, but the
underlying idea is easy to describe: Namely, focus on integers for which () ln n k + 1/2, where k is a positive integer.
In this case, cos(2 ln n) cos(2(k + 1/2)) = 1, so the exponent of n in the given series will be approximately 1.
Thus, if one can show that () holds for long enough intervals of ns, then the series behaves like a harmonic series over
these stretches and therefore diverges.
To make this precise, define the intervals Ik , for k = 1, 2, 3, . . . , by
h 
Ik = ek+1/2 , ek+1/2+1/(2k) .

Note that these intervals are disjoint. We will show that the given sum restricted
P to the interval Ik is c/k, where c
is a positive constant. Hence the entire series is bounded from below by k=1 c/k, a divergent series.
To prove this, note first that
1 1 1
n Ik ek+ 2 n < ek+ 2 + 2k
1 1 1
k + ln n < k + +
2   2 2k   
1 1 1
= cos 2 k + cos(2 ln n) < cos 2 k + +
2 2 2k
  
1
cos() cos(2 ln n) < cos 1 +
k

= 1 cos(2 ln n) < 1 + ,
k
where the latter inequality follows from the bound

cos((1 + x)) cos() + x max | sin((1 + t))| 1 + x.


0tx

Also, if n Ik , then
1 1
ln n < k + + k + 1 2k,
2 2k
1 e(ln n)/k e2
= .
n1+/k n n

2
It follows that
X 1 X e2
> .
nIk
n2+cos( ln n) nIk
n
Z 
2 1 1
e dt k+1/2 ,
I t e
 k 
2 k+1/2+1/k k+1/2 1
=e ln(e ) ln(e ) k+1/2 ,
e
 
1
e2 ek .
k

Summing the latter expression over all k gives a divergent series. Hence the given series diverges as well.

6. Let Gn denote the geometric mean of the n binomial coefficients n1 , n2 , . . . , nn .


  

Prove that the limit limn n Gn exists, and find its value. (You may only use the definition of binomial coefficients
and standard results from calculus, but not, for example, asymptotic formulas for binomial coefficients or factorials.)
1/n
Solution. We will show that limn Gn = e. Using the definition of the binomial coefficients, we get
n   n
Y n Y n! n!n
Gnn = = = 2 2
k k!(n k)! 1! 2! . . . n!2
k=1 k=1
1n 2n nn
= 2n 2n2 2n4 .
1 2 3 . . . (n 1)4 n2

Note that, for each i {1, 2, . . . , n}, the factor i occurs exactly n times in the numerator and 2(n i + 1) times in the
denominator. Hence,
n
X
ln Gnn = (ln i)(n 2(n i + 1))
i=1
Xn n
X
(1) = (ln i)(2i n) 2 ln i.
i=1 i=1

Also,
n  
X n(n + 1) 2
(2) (ln n)(2i n) = (ln n) 2 n = n ln n.
i=1
2

Subtracting (2) from (1) gives


n
X n
X
ln Gnn = (ln i ln n)(2i n) 2 ln i + n ln n.
i=1 i=1

Dividing by n2 we get
n   
1 1X i i
(3) ln G1/n
n = ln G n
n = ln 2 1 + R(n),
n2 n i=1 n n

where
n
ln n 2 X ln n
R(n) = 2 ln i + .
n n i=1 n
Since
n
ln n 2 X 3 ln n
|R(n)| + 2 ln n ,
n n i=1 n

3
1/n
we have limn R(n) = 0, so the term R(n) has no effect on the limit of ln Gn , and we can therefore ignore it.
R1
The remaining term on the right of (3) can be interpreted as a Riemann sum for the integral 0 (ln x)(2x 1)dx, and
evaluating this integral gives the desired limit:
n   
1X i i
lim ln G1/n
n = lim ln 2 1
n n n n n
i=1
Z 1
= (ln x)(2x 1)dx
0
1 Z 1 1
= (x2 x) ln x (x2 x) dx

0 0 x
Z 1
1
= (x 1)dx = .
0 2
1/n
Exponentiating, we get limn Gn = e1/2 , as claimed.

You might also like